Vous êtes sur la page 1sur 6

Bob Lutz

MATH 138
Problem Set II
02/17/2012

Problems 1(a)(b),4(i)(ii)(iii),5,6(a)(b),9(a)(b)
Problem 1.
(a) Suppose that X is a vector space with an additive norm, prove that X is a metric
linear space, that is scalar multiplication is jointly continuous, provided:
i. for each scalar , the map f 7 f is continuous at f = 0;
ii. for each vector f , the map 7 f is continuous at = 0;
iii. the scalar multiplication map (, f ) 7 f is continuous at (0, 0).
(b) Prove that L p is a metric linear space for all 0 < p < 1.
(a) Suppose that (i) - (iii) hold. Fix > 0, and let 0 R and f 0 X . By (i) there
exists (0 ) > 0 such that if k f k < (0 ), then k0 f k < 3 . Likewise, by (ii) there exists
( f 0 ) > 0 such that if || < ( f 0 ), then k f 0 k < 3 . By (iii), there exists 0 > 0 such
that if || < 0 and k f k < 0 , then k f k < 3 . Thus if | 0 | < min{( f 0 ), 0 } and
k f f 0 k < min{(0 ), 0 }, then

k0 f 0 f k = k( 0 )( f 0 f ) + f 0 (0 ) + 0 ( f 0 f )k
k( 0 )( f f 0 )k + k f 0 ( 0 )k + k0 ( f f 0 )k
< 3 + 3 + 3 = ,
proving that (, f ) 7 f is continuous.

(b) Suppose that p (0, 1). Given f L p which is nonzero on a set of positive mea1/p
sure, if || < k f k p
, then k f k p = || p k f k p < , so (a)(i) holds. Given 6= 0, if
k f k p || p , then k f k p = || p k f k p < , so (a)(ii) holds as well. Finally, if ||, k f k p <
min{1/2 , 1/(2p) }, then k f k = || p k f k p < 1/2 1/2 = , so (a)(iii) holds. Hence L p is a
metric linear space by (a).

Problem 2. Suppose 1 p < . If f n , f L p and f n f a.e., then k f n f k p 0 iff
k f n k p k f k p . (Use the generalized dominated convergence
which
R theorem
R
R statesR that
1
if f n , gn , f , g L , f n f and gn g a.e., | f n | gn , and gn g, then f n f .)
Suppose that k f n f k p 0, so that k f n f k p |k f n k p k f k p | 0 as well, whence
k f n k p k f k p . Suppose on the other hand that k f n k p k f k p . Note that

| f n f | p [2 max{| f n |, | f |}] p 2 p (| f n | p + | f | p ) = gn ,
1

and that | f n f | p 0 and gn 2 p+1 | f | p = g a.e. Since | f n f | p , gn , 0, g L1 and


Z
Z


1/p
1/p
1/p
2 p+1 k f k p = g,
gn = 2 p k f n k p + k f k p
it follows from the generalized dominated convergence theorem that
whence k f f n k p 0 as well.

| f f n | p 0,


Problem 5.
(a) If is -finite and f n f a.e., there exist measurable E1 , E2 , . . . X such that
S
(( 1 Ej )c ) = 0 and f n f uniformly on each Ej .
(b) Let be counting measure on N. Then f n f in measure iff f n f uniformly.
(a) Since is -finite, there exists a cover { Xk }1 of X such that ( X j ) < for all j N.
Since subsets of null sets are null, for all j N we have f n f for a.e. x X j . By
Egoroffs theorem, there exists a measurable1 set Fk (`) Xk such that ( Fj (k )c ) k1
S
and f n f uniformly on Fj (k). Let Ej (`) = `k=1 Fj (k ). Observe that { Ej (`)c }
`=1 is nested
downward, f n f uniformly on Ej (`), and
( Ej (`)c ) =

`
\

!
Fj (k)c

k =1

Thus Ej =

`=1

1
( Fj (`)c ) .
`

Ej (`) is measurable, f n f uniformly on Ej , and


( Ecj ) =

!
Ej (`)c

= lim ( Ej (`)c ) = 0.
`

`=1

by continuity from above. By countable subadditivty, ((


which concludes the proof.

S
1

Ej )c ) = (

T c
1 E j ) = 0,


(b) If f n f in measure, then given > 0 we have ({ x : | f n ( x ) f ( x )| > }) 0


as n . Since assumes natural values, there exists N N such that if n N, then
({ x : | f n ( x ) f ( x )| } = 0. But then { x : | f n ( x ) f ( x )| } = , whence f n f
uniformly. Suppose, on the other hand, that f n f uniformly. Given > 0 there exists
N N such that if n N, then { x : | f n ( x ) f ( x )| } = . But then for all n N we
have ({ x : | f n ( x ) f ( x )| > }) = 0, whence f n f in measure.


1 Folland

omits this distinction [p. 62], but Rudin does not [p. 73].

Problem 6.
(a) Suppose | f n | g L1 and f n f in measure.
R
R
i.
f = lim f n .
ii. f n f in L1 .
(b) In Egoroffs theorem, the hypothesis ( X ) < can be replaced by | f n | g for
all n, where g L1 ().

R
R
R
R
(a) Note that (i) follows from (ii), since | f f n | = | ( f f n )| | f f n |. Thus
1
it suffices to prove (ii). Let > 0 be
that
R given. Since g L , there exists E X such

( E) < , supxE | g( x )| < , and Ec g < 4 . Let An = { x : | f n ( x ) f ( x )| 3(E) }, and


note that lim ( An ) = 0. Since | f n f | | f n | + | f | 2g, we have
lim

| fn f |

| f n f | + lim

Ec

Z
E An

| f n f | + lim

Z
E\ An

| fn f |

2g + lim ( An ) sup 2g( x ) + ( E) sup | f n ( x ) f ( x )|

Ec

xE

+ ( E)
2

3( E)

x E\ An

< .

Hence f n f in L1 , as desired.

(b) In the proof of Egoroffs theorem [Folland, p. 62], we needed the former hypothesis
to invoke continuity from above on the sequence { En (k )}. Thus it suffices to show that if
| f n | g for all n N, then ( E1 (k)) < .
First, recall that
n
o
[
En (k) =
x : | f m ( x ) f ( x )| k1 .
m=n

Since | f m f | | f m | + | f | 2g for all m N, we have


n
o n
o
x : | f m ( x ) f ( x )| k1 x : 2g( x ) k1 .
Thus
E1 (k ) =

n
[

x : | f m ( x ) f ( x )| k1

n
o
x : 2g( x ) k1 .

m =1

Since g L1 (), we have


n

x : 2g( x ) k

Hence
( E1 (k ))

n

o

2k

x : 2g( x ) k

as desired.

g < .
o

< ,


Problem 8.
(a) Suppose ( X ) < . If f and g are complex-valued measurable functions on X,
define
Z
| f g|
( f , g) =
d.
1 + | f g|
Then is a metric on the space of measurable functions if we identify functions
that are equal a.e., and f n f with respect to this metric iff f n f in measure.
Prove this by showing that

k f k0 =

|f|
d
1 + |f|

is an additive norm. Also show that a sequence is Cauchy in measure if and only
if it is Cauchy in the metric.
(b) Show that the space of measurable functions is a metric linear space under the
additive norm k f k0 .
(a) Since | f | = | f |, we have k f k0 = k f k0 . To prove the triangle inequality for k k0 ,
let f , g be measurable. Since integration is linear, it suffices to show that

k f + g k0 =

|f|
| g|
| f + g|

+
.
1 + | f + g|
1 + | f | 1 + | g|

After multiplying through and canceling terms, this simplifies to the inequality

| f + g| | f | + | g| + | f || g|(2 + | f + g|),
which holds due to the absolute value triangle inequality. Thus k k0 is an additive norm.
Let h be a measurable function, and notice that
( f , h) = k f hk0 k f gk0 + k g hk0 = ( f , g) + ( g, h),
proving the triangle inequality for . In addition, since the integrand in the definition of
is nonnegative, ( f , g) = 0 if and only if the numerator of the integrand is zero a.e.,
which occurs if and only if f = g a.e. Hence if we identify functions that are equal a.e.,
is nondegenerate and therefore a metric.
Suppose that f n f in measure. Given > 0, let E = { x : | f n ( x ) f ( x )| > }. Since
the map 7 1+ is increasing and bounded above by 1, we have

| f n ( x ) f ( x )|
1
1 + | f n ( x ) f ( x )|
and

| f n ( x ) f ( x )|

<
1 + | f n ( x ) f ( x )|
1+
4

for all x E,

for all x Ec .

Thus
( f n , f ) =

Z
E

| f n ( x ) f ( x )|
d +
1 + | f n ( x ) f ( x )|

Z
E

d +

Z
Ec

Z
Ec

| f n ( x ) f ( x )|
d
1 + | f n ( x ) f ( x )|

= ( E) + ( X ).
But f n f in measure, so ( E) 0 as n . Since ( X ) < , it follows that
( f n , f ) 0.
Suppose on the other hand that ( f n , f ) 0. Then we have


Z
Z

| f n ( x ) f ( x )|
d
d =
( E ),
( f n , f ) =
1 + | f n ( x ) f ( x )|
1+
E 1+
whence ( E) 0, proving that f n f in measure. Hence f n f with respect to if and
only if f n f in measure. If instead we set E = { x : | f n ( x ) f m ( x )| > } and replace f
with f m , the preceding argument yields the Cauchy result as well.

(b) Let > 0 and R be given. Suppose that || 1, and let f be measurable such
that k f k0 < . Then

k f k0 =

||| f |
d
1 + ||| f |

|f|
d = k f k0 < .
1 + |f|

(1)

Suppose that || > 1, and let f be such that k f k0 < ||1 . Then

k f k0 =

||| f |
d
1 + ||| f |

||| f |
d = ||k f k0 < ,
1 + |f|

(2)

proving hypothesis (i) of Problem 1(a). Next, let f be a given measurable function. If
k f k0 < , then let || 1, so that (1) follows. Otherwise, let || < k f k01 , so that (2)
follows. Thus 1(a)(ii) holds as well. Finally, if || < 1 and k f k0 < , then (1) follows, so
1(a)(iii) holds. Hence the space of measurable functions is a metric linear space.

Problem 10. On Rk find a sequence with f n 0 in L1 and a.e. but not almost uniformly.
Let An = ik=1 (n, n + n1/k ] for all n N, and consider the sequence { f n } defined by
f n = En . Note that for each n N, since An is a half-open rectangle, we have
k

k f n k1 = ( A n ) = m
i =1



1
n, n +
k
n



k
n

k

1
0,
n

as n ,

whence f n 0 in L1 . Moreover, given x = ( x1 , x2 , . . . , xk ), if n > max{ xi : i = 1, . . . , k },


then f n (x) = 0. Thus f n 0 pointwise, proving that f n 0 a.e. Note that if f n 0
5

uniformly on E, then there exists N N such that n N implies An E = . Since the


An are mutually disjoint, monotonicity and countable additivity imply
!

[
1
= .
( Ec ) m
An =
n
n= N
n= N
Hence f n does not converge almost uniformly.

Vous aimerez peut-être aussi